4
$\begingroup$

Denote $g(n)=\sum_{a,b=1}^n\gcd(a,b)$, can we prove that $$g(n)=\frac6{\pi^2}n^2\ln n+Cn^2+O(n\ln n)$$, where $C=-\frac12+\frac{6}{\pi^2}(-\frac12+\gamma-\ln(2\pi)+12\ln A),$ where $\gamma$ denotes Euler's constant and $A$ denotes Glaisher's constant?
Failed Attempt
$$g(n)=\sum_{k=1}^n\varphi(k)\left[\frac nk\right]^2\\ =\sum_{k=1}^n\varphi(k)\left[\frac nk\right]^2=\sum_{k=1}^n\varphi(k)\left(\frac nk+O(1)\right)^2\\=n^2\sum_{k=1}^n\frac{\varphi(k)}{k^2}+O\left(n\sum_{k=1}^n\frac{\varphi(k)}k\right)\\ =\frac{6}{\pi^2}n^2\ln n+O(n^2)$$ Closer Attempt to $O(n^{3/2}\ln n)$
$$g(x)=2\sum_{k\le n\le x}\gcd(n,k)-\frac12x^2+O(x)\\ =2\sum_{dl\le x}d\varphi(l)-\frac12x^2+O(x)\\ =2\sum_{d\le \sqrt x}d\sum_{l\le x/d}\varphi(l)+ 2\sum_{l\le \sqrt x}\varphi(l)\sum_{d\le x/l}d-2\left(\sum_{d\le \sqrt x}\varphi(d)\right)\left(\sum_{l\le \sqrt x}l\right)-\frac12x^2+O(x)\\ =2\sum_{d\le \sqrt x}d\left(\frac3{\pi^2}\frac{x^2}{d^2}+O\left(\frac xd\ln x\right)\right)+2\sum_{l\le \sqrt x}\varphi(l)\frac{x^2}{2l^2}+O\left(\frac xl\right)-2\left(\frac 3{\pi^2}x+O(\sqrt x\ln x)\right)\left(\frac x2+O(\sqrt x)\right)\\ =\frac{6}{\pi^2}x^2(\ln\sqrt x+\gamma)+\frac{6}{\pi^2}x^2(\ln\sqrt x+\gamma)-\frac{36}{\pi^4}\zeta'(2)x^2-\left(\frac 3{\pi^2}+\frac12\right)x^2+O(x^{3/2}\ln x)\\ =\frac6{\pi^2}x^2\ln x+Cx^2+O(x^{3/2}\ln x)$$ How can I improve the big-$O$ term?

$\endgroup$

1 Answer 1

12
$\begingroup$

The asymptotic you want does not hold just because the "last-term fluctuation" $$ g(n)-g(n-1) = 2\sum_{a=1}^n \gcd(a,n)-n $$ is too large. Indeed, denoting the sum in the right-hand side by $\sigma(n)$, we have $$ \sigma(n) =\sum_{d\mid n} d\varphi(n/d) = n \sum_{d\mid n} \prod_{p\mid(n/d)} \Big(1-\frac1p\Big) = n \sum_{d\mid n} \prod_{p\mid d} \Big(1-\frac1p\Big). $$

If now $n=p_1\dotsb p_k$ is the product of the first $k$ primes, then $$ \sigma(n) = \Big(2-\frac1{p_1}\Big)\dotsb\Big(2-\frac1{p_k}\Big)\,n \ge (3/2)^k n $$ while, using the prime number theorem, it is not difficult to see that $k\ge c\ln n/\ln\ln n$ with an absolute constant $c>0$; hence, $$ \sigma(n) \ge \exp(c'\ln n/\ln\ln n)\,n,\ n=p_1\dotsb p_k. $$ On the other hand, for $n$ prime we have $$ \sigma(n) = 2n-1. $$ It follows that one cannot get an asymptotic for $g(n)$ with the remainder term better than $O(\exp(c'\ln n/\ln\ln n)\,n)$.

$\endgroup$

Your Answer

By clicking “Post Your Answer”, you agree to our terms of service and acknowledge you have read our privacy policy.

Not the answer you're looking for? Browse other questions tagged or ask your own question.